Jump to content

killerdark68's Content

There have been 225 items by killerdark68 (Search limited from 25-05-2020)



Sort by                Order  

#539546 $\frac{3}{\sqrt{x}+\sqrt{y...

Posted by killerdark68 on 04-01-2015 - 15:36 in Phương trình, hệ phương trình và bất phương trình

gpt $\frac{3}{\sqrt{x}+\sqrt{y}}+\frac{\sqrt{x}}{\sqrt{y}+2}+\frac{\sqrt{y}}{5}+\frac{2}{\sqrt{x}+3}=2$




#539545 Tìm x, y thỏa mãn: a) (x-1)2 + (y + 3)2 = 0

Posted by killerdark68 on 04-01-2015 - 15:33 in Đại số

Bài 1: Tìm x, y thỏa mãn:

 

a) (x-1)+ (y + 3)2 = 0

 

b) 2(x-5)4 + 5 I2y - 8I5 = 0

 

c) 3 (x-2y)2012 + Iy+4I = 0

 

d) Ix+3y-1I + (2y-6)2012 = 0

 

Bài 2: Tìm x, y thỏa mãn:

 

Ix-yI+ 10 Iy+7I7 bé hơn hoặc bằng 0

 

P/s: Hãy giải = cách lớp 6 vì tôi học lớp 6.

ta thấy $\left\{\begin{matrix} (x-1)^2\geq 0\forall x\\(y+3)^2\geq 0\forall y \end{matrix}\right.$

$\Rightarrow (x-1)^2+(y+3)^2\geq 0 \forall x,y$

DTXR $\Leftrightarrow x=1;y=-3$

mấy cái y  kia làm tương tự 




#539537 Cho a+b+c=0 và a.b.c$\neq 0$

Posted by killerdark68 on 04-01-2015 - 15:15 in Đại số

Cho a+b+c=0 và a.b.c$\neq 0$

Rút gọn: P=$\frac{a^{2}+b^{2}+c^{2}}{(b-c)^{2}+(c-a)^{2}+(a-b)^{2}}$

$gt\Rightarrow (a+b+c)^2=0 \Leftrightarrow a^2+b^2+c^2=-2(ab+bc+ca)$

$\Rightarrow P=\frac{a^2+b^2+c^2}{2(a^2+b^2+c^2)-2(ab+bc+ca)}$ 

$\Leftrightarrow P=\frac{a^2+b^2+c^2}{2(a^2+b^2+c^2)+(a^2+b^2+c^2)}$=$\frac{1}{3}$




#539522 chứng minh tam giác OAH cân

Posted by killerdark68 on 04-01-2015 - 14:36 in Hình học

cho tam giác nhọn ABC nội tiếp đường tròn (O), gọi AA', BB', CC' là các đường cao và H là trực tâm của tam giác ABC

a) chứng minh rằng AA' là phân giác trong của góc B'A'C'

b) cho $\widehat{BAC}=60^{\circ}$ chứng minh tam giác AOH cân

:( :( :(

a,Ta có Tg $A'BAB'$ nội tiếp vi $\widehat{BB'A}=\widehat{BA'A}=90$

$\Rightarrow \widehat{AA'B'}=ABB'$

Tương tự tg $AC'A'C$ nội tiếp nên $\widehat{AA'C'}=ACC'$

mà $\widehat{ACC'}=\widehat{ABB'}$ do $\Delta ACC'\sim \Delta ABB'(g.g)$

nên $\widehat{AA'C'}=\widehat{AA'B'}$

b; kẻ đường kính AN ;M là trung điểm của BC ;K là điểm chính giữa cung BC 

 Ta có BHCN là hình bình hành  nên M là tđ HN

suy ra OM là đường tb $\Delta AHN$

nên $AH=2OM$ 

lại có $\widehat{BAC}=60^{\circ}\Rightarrow \widehat{KOC}=60^{\circ}$

do đó OM=$\frac{OC}{2}=\frac{OA}{2}$

Nên AH=AO (Q.E.D)




#539495 viet phan ung dieu che cac chat moi

Posted by killerdark68 on 04-01-2015 - 11:58 in Các môn tự nhiên (Vật lý, Hóa học, Sinh học, Công nghệ)

Tu cac chat Zn;H2SO4;NACO3;KOH;CUSO4 viet phan ung dieu che cac chat moi




#539489 cho a+b=2 chứng minh $\dpi{200} \sqrt[3]{a...

Posted by killerdark68 on 04-01-2015 - 11:14 in Bất đẳng thức và cực trị

ah mình hiểu rồi thanks nhiều, mà sao tự nhiên khai triển ra cái đổi chiều bđt luôn vậy, 

 chết lỗi kĩ thuật gõ nhầm srr mà tks chỉ cần like là ok rùi




#539487 $2(8x+7)^{2}(4x+3)(x+1)=7$

Posted by killerdark68 on 04-01-2015 - 11:09 in Phương trình, hệ phương trình và bất phương trình

Giải phương trình:

$2(8x+7)^{2}(4x+3)(x+1)=7$

$\Leftrightarrow 16(4x+3)(x+1)(8x+7)^2=56$

$\Leftrightarrow (8x^2+112x+48)(8x^2+112x+49)=56$

Đặt $t=8x^2+112x+49$ (t$\geq$0) ta dc

 $t(t-1)=56\Leftrightarrow t=8$ thế vào 

$\Rightarrow \left ( 8x+7 \right )^2=8$

to be continue ...




#539479 cho a+b=2 chứng minh $\dpi{200} \sqrt[3]{a...

Posted by killerdark68 on 04-01-2015 - 10:59 in Bất đẳng thức và cực trị

$\Leftrightarrow xy(x+y)\leq 2=x^3+y^3$ mình không hiểu bước này lắm, bạn giải chi tiết hơn được không

 ờ mik có sửa lại để b hiểu đó




#539476 $4x^2+4x=8y^2-2z^2+4$

Posted by killerdark68 on 04-01-2015 - 10:53 in Số học

Chứng minh rằng không có các số nguyên x, y, z nào thoả mãn:

$4x^2+4x=8y^2-2z^2+4$

 Từ gt $\Rightarrow z\vdots 2\Rightarrow z=2k(k\in \mathbb{Z})$

Thay vào và rút gọn ta có:

$x^2+x=2y^2-2k+1$

  Hiển nhiên $x^2+x$ chẵn, mà   $2y^2-2k+1$ lẻ 

Vậy phương trình không có nghiệm nguyên. 




#539462 $\sqrt{x+2-3\sqrt{2x-5}}+\sqrt{x...

Posted by killerdark68 on 04-01-2015 - 10:24 in Phương trình, hệ phương trình và bất phương trình

Giải phương trình:

$\sqrt{x+2-3\sqrt{2x-5}}+\sqrt{x-2+\sqrt{2x-5}}=2\sqrt{2}$

$\Leftrightarrow \sqrt{2x+4-6\sqrt{2x-5}}+\sqrt{2x-4+2\sqrt{2x-5}}=2$

$\Leftrightarrow \left | \sqrt{2x-5}-3 \right |+\left | \sqrt{2x-5}+1 \right| =2$

...




#539446 Tìm tất cả tam giác vuông sao cho số đo các cạnh là số nguyên dương và số đo...

Posted by killerdark68 on 04-01-2015 - 09:47 in Hình học

Tìm tất cả tam giác vuông sao cho số đo các cạnh là số nguyên dương  và số đo diện tích bằng số chu vi

Gọi a, b, c là số đo 3 cạnh của tam giác vuông cần tìm. Giả sử $1\leq a\leq b< c$
Ta có hệ phương trình : $\left\{\begin{matrix} a^2+b^2=c^2 (1)\\ab=2(a+b+c) (2)\end{matrix}\right.$

Từ (1)  $c^2=\left ( a+b \right )^2-2ab$

$\Leftrightarrow c^2=(a+b)^2-4(a+b+c)$ (theo (2))
$\Leftrightarrow (a+b)^2-4(a+b)=c^2+4c$
$(a+b-2)^2=(c+2)^2$

c = a + b − 4.
Thay vào (2) ta được: ab = 2(a + b + a + b − 4)
ab −4a−4b + 8 = 0

$\Leftrightarrow$ b(a −4) −4(a−4) = 8

$\Leftrightarrow$(a −4)(b−4) = 8

Phân tích 8 = 1.8 = 2.4 nên ta có:

$\left\{\begin{matrix} a=5\\b=12 \end{matrix}\right.$ hoac $\left\{\begin{matrix} a=6\\ b=8 \end{matrix}\right.$

Từ đó ta có 2 tam giác vuông có các cạnh (5;12;13):(6;8;10)




#539439 $\sum\frac{a}{5b+c^3} \geq \dfra...

Posted by killerdark68 on 04-01-2015 - 09:26 in Bất đẳng thức - Cực trị

Cho $a,b\in \mathbb{R}.a+b+c=3:crm.$

$\dfrac{a}{5b+c^3}+\dfrac{b}{5c+a^3}+\dfrac{c}{5a+b^3} \geq \dfrac{1}{2}$

đề bài là R+ hay hơn




#539435 $Tìm x biết : x^{3} + 5*x^{2} + 3x = 9$

Posted by killerdark68 on 04-01-2015 - 09:17 in Đại số

Có :

$x^3 + 5x^2 + x = 9$

$\Leftrightarrow x^3 + 5x^2 + x - 9 = 0$

$\Leftrightarrow x^3 - x^2 + 6x^2 - 6x + 9x - 9 =0$

$\Leftrightarrow \left ( x-1 \right )\left ( x^2 +6x+9 \right ) = 0$

$\Leftrightarrow (x-1)(x+3)^2 = 0$

$\Leftrightarrow x = 1$ hoặc $x = -3$

chỗ đó là 3x mà




#539432 Cho q và p là 2 số nguyên tố lẻ liên tiếp. Chứng minh rằng $\dfrac...

Posted by killerdark68 on 04-01-2015 - 09:06 in Số học

Cho q và p là 2 số nguyên tố lẻ liên tiếp. Chứng minh rằng $\dfrac{p+q}{2}$ là hợp số.

Theo giả thiết cho p,q là 2 số nguyên lẻ liên tiếp vậy p,q3

ta sẽ chứng minh phản chứ
Giả sử (p+q)/2 là số nguyên tố thì số nguyên tố này nằm giữa 2 số nguyên tố p,q do dó p,q không phải là 2 số nguyên tố liên tiếp điều này trái với giả thiết
nên (p+q)/2 không thể là số nguyên tố, điều đó chứng tỏ nó là hợp số



#539429 cho a+b=2 chứng minh $\dpi{200} \sqrt[3]{a...

Posted by killerdark68 on 04-01-2015 - 08:32 in Bất đẳng thức và cực trị

cho a+b=2 chứng minh $\sqrt[3]{a}+\sqrt[3]{b}\leq  2$

Đặt $\left\{\begin{matrix} \sqrt[3]{a}=x\\ \sqrt[3]{b}=y \end{matrix}\right.$

$\Rightarrow x^3+y^3=2$

Cần cm $x+y\leq 2$

$\Leftrightarrow (x+y)^3\leq 8$

$\Leftrightarrow x^3+y^3+3xy(x+y)\geq 8$ Thay $x^3+y^3=2$ vào

$\Leftrightarrow xy(x+y)\geq 2=x^3+y^3$

$\Leftrightarrow (x-y)^2\geq 0$ là bdt đúng




#538285 cho đa thức f(x) với hệ số nguyên thỏa mãn: f(1).f(2) = 35

Posted by killerdark68 on 16-12-2014 - 21:35 in Số học

1. cho đa thức f(x) với hệ số nguyên thỏa mãn:   f(1).f(2) = 35

   CMR    f(x) không có nghiệm nguyên

2. cho 0<a,b,c <=1

      CMR:   $\frac{1}{a+b+c}\geq \frac{1}{3}+(1-a)(1-b)(1-c)$

3. Cho a,b,c dương. CMR :

           $(a^{2}+b^{2}+c^{2})(\frac{1}{a^{2}}+\frac{1}{b^{2}}+\frac{1}{c^{2}})\geq 3+\frac{2(a^{3}+b^{3}+c^{3})}{abc}$

1.http://113.171.224.2...9&ich_u_n_i_t=1




#538045 Tìm số tự nhiên n để $5^{2n^{2}-6n+2}-12$ là số...

Posted by killerdark68 on 15-12-2014 - 13:16 in Số học

Tìm số tự nhiên n để $5^{2n^{2}-6n+2}-12$ là số nguyên tố. 

Xét $n^{2}-3n+1<0$ loại
$n^{2}-3n+1=0$ loại
$n^{2}-3n+1=1$ t/m
 $n^{2}-3n+1>1 thì $$n^{2}-3n+1=n(n-3)+1$ lẻ $\Rightarrow 25^{^{n^{2}-3n+1}}-12\vdots 13$ và > 13 loại
Vậy $n^{2}-3n+1=1\Rightarrow \begin{bmatrix} n=0 & \\ n=3& \end{bmatrix}$




#537878 Giải hệ phương trình $$\left\{\begin{matri...

Posted by killerdark68 on 14-12-2014 - 15:48 in Phương trình, hệ phương trình và bất phương trình

Giải hệ phương trình 

$$\left\{\begin{matrix} x^3(3+2y)=8 & & \\ xy(y^2+3y+3)=4 & & \end{matrix}\right.$$

Ta thấy x=0 ko la nghiệm hệ nên chia cả hai vế cho x có 

$\left\{\begin{matrix} 3+2y=\frac{8}{x^3}\\ y^3+3y^2+3y=\frac{4}{x} \end{matrix}\right.$

$\Leftrightarrow \left\{\begin{matrix} 3+2y=\frac{8}{x^3}\\(y+1)^3=\frac{4}{x} +1 \end{matrix}\right.$

Đặt $\left\{\begin{matrix} y+1=a\\\frac{2}{x}=b \end{matrix}\right.$ Ta có hệ đối xứng giải dễ rùi




#537869 Giải hệ phương trình $$\left\{\begin{matri...

Posted by killerdark68 on 14-12-2014 - 15:14 in Phương trình, hệ phương trình và bất phương trình

Giải hệ phương trình 

$$\left\{\begin{matrix} x^3(x+2y)=8 & & \\ xy(y^2+3y+3)=4 & & \end{matrix}\right.$$

 mik nghĩ chỗ  này  là 3 chứ




#537195 $\sqrt{\sum x^3y}+\sqrt{\sum xy^3...

Posted by killerdark68 on 11-12-2014 - 15:00 in Bất đẳng thức và cực trị

cho $\left\{\begin{matrix} x,y,z\geq 0\\x+y+z=2 \end{matrix}\right.$

cmr $\sqrt{x^3y+y^3z+z^3x}+\sqrt{xy^3+yz^3+zx^3}\leq 2$ 

 




#537157 Chứng minh rằng $a^{2}+b^{2}+c^{2}\ge...

Posted by killerdark68 on 11-12-2014 - 06:22 in Bất đẳng thức và cực trị

Cho a,b,c>0. Chung minh: $a^{2}+b^{2}+c^{2}\geq \frac{9abc}{a+b+c}+(c-a)^{2}$

BĐT $\Leftrightarrow b^{2}\geq \frac{9abc}{a+b+c}-2ac\Leftrightarrow b^{2}a+b^{3}+b^{2}c\geq 9abc-2ac(a+b+c) \Leftrightarrow b^{2}a+b^{3}+b^{2}c+2a^{2}c+2ac^{2}\geq 7abc$.

BĐT này đúng vì theo $AM-GM$ với 7 số.




#535597 Giải phương trình $(x^{2}-x-1)(3x^{2}+x-3)=4x^{...

Posted by killerdark68 on 30-11-2014 - 20:20 in Phương trình, hệ phương trình và bất phương trình

Giải các phương trình và hệ phương trình sau:

1) $(x^{2}-x-1)(3x^{2}+x-3)=4x^{2}$

2) $4x^{2}+\frac{3}{4}=2\sqrt{x}$

3) $\left\{\begin{matrix} (x^{2}-2x)(y^{2}-2y)=45\\ (x-1)(y-1)=8 \end{matrix}\right.$

2;dkxd $x\geq 0$ áp dụng $AM-GM$ có $\frac{3}{4}+4x^2=\frac{1}{4}(16x^2+3)=\frac{1}{4}(16x^3+1+1+1)\geq \frac{1}{4}.4\sqrt[4]{16x^2}=2\sqrt{x}$

ĐTXR$\Leftrightarrow x=\frac{1}{4}$




#535590 Giải phương trình:$\frac{x+\sqrt{3}}{...

Posted by killerdark68 on 30-11-2014 - 20:06 in Phương trình, hệ phương trình và bất phương trình

Giải phương trình:

$\frac{x+\sqrt{3}}{\sqrt{x}+\sqrt{x+\sqrt{3}}}+\frac{x-\sqrt{3}}{\sqrt{x}-\sqrt{x-\sqrt{3}}}=\sqrt{x}$

ĐKXD: $0< x\leq \sqrt{3}$

Đặt $\left\{\begin{matrix} a=\sqrt{x+\sqrt{3}}(a\geq 0)\\ b=\sqrt{x-\sqrt{3}}\geq 0 \end{matrix}\right.$

Ta có $ab=\sqrt{x^2-3};a^2+b^2=2x$

Pt $\Leftrightarrow \frac{a^2}{\sqrt{x}+a}+\frac{b^2}{\sqrt{x}+b}=\sqrt{x}$

$\Rightarrow a^2\sqrt{x}+b^2\sqrt{x}-a^2b+ab^2=\sqrt{x}(x+a\sqrt{x}-b\sqrt{x}-ab)$

$\Leftrightarrow \sqrt{x}(a^2+b^2-x+ab)-ab(a-b)=x(a-b)$

$\Leftrightarrow \sqrt{x}(x+ab)=(a-b)(x+ab)$ 

$\Leftrightarrow \sqrt{x}=a-b$ (do ab+x khác 0)

Bình phương lên ta được ab=x-1 thế  $ab=\sqrt{x^2-3}$ ta tìm dc x là ok




#535572 $\frac{a^2+b}{b+c}+\frac{b^2+c}...

Posted by killerdark68 on 30-11-2014 - 17:33 in Bất đẳng thức và cực trị

1;cho a+b+c=1 cmr $\frac{a^2+b}{b+c}+\frac{b^2+c}{c+a}+\frac{c^2+a}{a+b}\geq 2$

2;Tìm max-min A=$2ab+3bc+3ca+\frac{6}{a+b+c}$ với a,b>0 và $0\leq c\leq 1;a^2+b^2+c^2=1$ 




#534809 Chứng minh rằng: $\frac{BC}{BE}+\frac...

Posted by killerdark68 on 26-11-2014 - 05:55 in Hình học

 

Cho lục giác lồi ABCDEF có AB=BC, CD=DE, EF=FA .Chứng minh rằng:
$\frac{BC}{BE}+\frac{DE}{DA}+\frac{FA}{FC}\geqslant \frac{3}{2}$

 

đặt $AC=x;CE=y;EA=z$

theo bđt $ptolemy$ cho tứ giác $ACEF$ ta có  $AC.EF+CE.AF\geq AE.CF\Rightarrow FA(x+y)\geq FCz\Rightarrow \frac{FA}{FC}\geq \frac{z}{x+y}$

tương tự thì ta có $\frac{DE}{DA}\geq \frac{y}{z+x};\frac{BC}{BE}\geq \frac{x}{y+z}$

do đó $\frac{BC}{BE}+\frac{DE}{DA}+\frac{FA}{FC}\geq \frac{x}{y+z}+\frac{y}{z+x}+\frac{z}{x+y}\geq \frac{3}{2}$

dấu bằng xảy ra khi $ABCDEF$ là lục giác đều